K
Khách

Hãy nhập câu hỏi của bạn vào đây, nếu là tài khoản VIP, bạn sẽ được ưu tiên trả lời.

NV
7 tháng 5 2021

\(4\le\left(\sqrt{x}+1\right)\left(\sqrt{y}+1\right)\le\dfrac{1}{4}\left(\sqrt{x}+\sqrt{y}+2\right)^2\)

\(\Rightarrow\sqrt{x}+\sqrt{y}+2\ge4\)

\(\Rightarrow2\le\sqrt{x}+\sqrt{y}\le\sqrt{2\left(x+y\right)}\Rightarrow x+y\ge2\)

\(\Rightarrow P\ge\dfrac{\left(x+y\right)^2}{x+y}=x+y\ge2\)

Dấu "=" xảy ra khi \(x=y=1\)

7 tháng 5 2021

Dạ có thể diễn đạt theo cách dễ hiểu cho đứa ngu lâu dốt bền như em được không ạ ? ._.

NV
1 tháng 9 2021

Đặt \(\left\{{}\begin{matrix}\sqrt{2x+3}=a\ge0\\\sqrt{y}=b\ge0\end{matrix}\right.\)

\(\Rightarrow b\left(b^2+1\right)-3a^2=\left(a^2+1\right)a-3b^2\)

\(\Rightarrow a^3-b^3+3a^2-3b^2+a-b=0\)

\(\Leftrightarrow\left(a-b\right)\left(a^2+ab+b^2\right)+\left(a-b\right)\left(3a+3b\right)+a-b=0\)

\(\Leftrightarrow\left(a-b\right)\left(a^2+ab+b^2+3a+3b+1\right)=0\)

\(\Leftrightarrow a=b\Rightarrow\sqrt{2x+3}=\sqrt{y}\)

\(\Rightarrow y=2x+3\)

\(\Rightarrow M=x\left(2x+3\right)+3\left(2x+3\right)-4x^2-3\) tới đây chắc chỉ cần bấm máy

11 tháng 9 2017

*)Maximize : Áp dụng BĐT Cauchy-Schwarz ta có:

\(VT^2\le\left(1+1\right)\left(x+1+y+1\right)=2\left(x+y+2\right)\)

Và \(VP^2=\left(\sqrt{2}\left(x+y\right)\right)^2=2\left(x+y\right)^2\)

\(\Rightarrow2\left(x+y\right)^2\le2\left(x+y+2\right)\)

\(\Rightarrow\left(x+y\right)^2-\left(x+y\right)-2\le0\)

\(\Rightarrow\left(x+y-2\right)\left(x+y+1\right)\le0\)

\(\Rightarrow-1\le P=x+y\le2\) 

Khi \(x=y=1\) thì \(P_{Max}=2\)

*)Minimize: Áp dụng BĐT Karamata ta có:

\(VT=\sqrt{2}\left(x+y\right)=\sqrt{x+1}+\sqrt{y+1}=VP\)

\(\ge\sqrt{0}+\sqrt{x+1+y+1}\)

\(\Rightarrow\sqrt{2}\left(x+y\right)\ge\sqrt{x+1+y+1}\)

\(\Rightarrow2\left(x+y\right)^2\ge\left(x+y\right)+2\)

\(\Rightarrow2\left(x+y\right)^2-\left(x+y\right)-2\ge0\)

\(\Rightarrow P=x+y\ge\frac{1+\sqrt{17}}{4}\)

Khi \(x=\frac{5+\sqrt{17}}{4};y=-1\) thì \(P_{Min}=\frac{1+\sqrt{17}}{4}\)

#Vỗ tay coi :))

11 tháng 9 2017

Thắng -_- ừ, hay lắm :))

\(\sqrt{xy}+\sqrt{x}+\sqrt{y}\ge3\)

ÁP DỤNG BĐT COSI
\(\sqrt{xy}+\sqrt{x}+\sqrt{y}\le\frac{x+y}{2}+\frac{x+1}{2}+\frac{y+1}{2}=x+y+1\ge3=>x+y\ge2\)

\(P\ge\frac{\left(x+y\right)^2}{x+y}=2\left(cosi\right)\) vậy min P=2 <=> x=y=1

12 tháng 5 2021

                      Bài làm :

Ta có :

\(\left(\sqrt{x}+1\right)\left(\sqrt{y}+1\right)\ge4\)

\(\Leftrightarrow\sqrt{xy}+\sqrt{y}+\sqrt{x}+1\ge4\)

\(\Leftrightarrow\sqrt{xy}+\sqrt{x}+\sqrt{y}\ge3\)

Áp dụng BĐT cosi cho các số không âm ; ta được :

\(3\le\sqrt{xy}+\sqrt{x}+\sqrt{y}\le\frac{x+y}{2}+\frac{x+1}{2}+\frac{y+1}{2}=x+y+1\)

\(\Rightarrow x+y\ge2\)

Ta có :

\(P=\frac{x^2}{y}+\frac{y^2}{x}\ge\frac{\left(x+y\right)^2}{x+y}=x+y\)

\(\Rightarrow P\ge2\)

Dấu "=" xảy ra khi x=y=1

Vậy MinP = 2 <=> x=y=1

10 tháng 8 2017

post từng câu một thôi bn nhìn mệt quá

13 tháng 3 2021

Theo giả thiết xy + yz + zx = 1 nên ta có: \(VT=\frac{1}{1+x^2}+\frac{1}{1+y^2}+\frac{1}{1+z^2}=\frac{1}{xy+yz+zx+x^2}+\frac{1}{xy+yz+zx+y^2}+\frac{1}{xy+yz+zx+z^2}=\frac{1}{\left(x+y\right)\left(x+z\right)}+\frac{1}{\left(y+x\right)\left(y+z\right)}+\frac{1}{\left(z+x\right)\left(z+y\right)}=\frac{2\left(x+y+z\right)}{\left(x+y\right)\left(y+z\right)\left(z+x\right)}\)Theo bất đẳng thức Cauchy-Schwarz: \(\left(\frac{x}{\sqrt{1+x^2}}+\frac{y}{\sqrt{1+y^2}}+\frac{z}{\sqrt{1+z^2}}\right)^2\le\left(x+y+z\right)\left(\frac{x}{1+x^2}+\frac{y}{1+y^2}+\frac{z}{1+z^2}\right)=\left(x+y+z\right)\left(\frac{x}{\left(x+y\right)\left(x+z\right)}+\frac{y}{\left(y+z\right)\left(y+x\right)}+\frac{z}{\left(z+x\right)\left(z+y\right)}\right)=\frac{2\left(x+y+z\right)\left(xy+yz+zx\right)}{\left(x+y\right)\left(y+z\right)\left(z+x\right)}=\frac{2\left(x+y+z\right)}{\left(x+y\right)\left(y+z\right)\left(z+x\right)}\)\(\Rightarrow\frac{2}{3}\left(\frac{x}{\sqrt{1+x^2}}+\frac{y}{\sqrt{1+y^2}}+\frac{z}{\sqrt{1+z^2}}\right)^3\le\frac{4\left(x+y+z\right)}{3\left(x+y\right)\left(y+z\right)\left(z+x\right)}\left(\frac{x}{\sqrt{1+x^2}}+\frac{y}{\sqrt{1+y^2}}+\frac{z}{\sqrt{1+z^2}}\right)\)Ta cần chứng minh: \(\frac{2\left(x+y+z\right)}{\left(x+y\right)\left(y+z\right)\left(z+x\right)}\ge\frac{4\left(x+y+z\right)}{3\left(x+y\right)\left(y+z\right)\left(z+x\right)}\left(\frac{x}{\sqrt{1+x^2}}+\frac{y}{\sqrt{1+y^2}}+\frac{z}{\sqrt{1+z^2}}\right)\)

hay \(\frac{x}{\sqrt{1+x^2}}+\frac{y}{\sqrt{1+y^2}}+\frac{z}{\sqrt{1+z^2}}\le\frac{3}{2}\)

Bất đẳng thức cuối đúng theo AM - GM do: \(\frac{x}{\sqrt{1+x^2}}+\frac{y}{\sqrt{1+y^2}}+\frac{z}{\sqrt{1+z^2}}=\sqrt{\frac{x}{x+y}.\frac{x}{x+z}}+\sqrt{\frac{y}{y+z}.\frac{y}{x+y}}+\sqrt{\frac{z}{z+x}.\frac{z}{z+y}}\le\frac{\left(\frac{x}{x+y}+\frac{x}{x+z}\right)+\left(\frac{y}{y+z}+\frac{y}{x+y}\right)+\left(\frac{z}{z+x}+\frac{z}{z+y}\right)}{2}=\frac{3}{2}\)Đẳng thức xảy ra khi \(x=y=z=\frac{1}{\sqrt{3}}\)